Ανισότητα!

Συντονιστές: achilleas, emouroukos, silouan

Άβαταρ μέλους
Διονύσιος Αδαμόπουλος
Δημοσιεύσεις: 807
Εγγραφή: Σάβ Μαρ 19, 2016 5:11 pm
Τοποθεσία: Πύργος Ηλείας

Ανισότητα!

#1

Μη αναγνωσμένη δημοσίευση από Διονύσιος Αδαμόπουλος » Κυρ Δεκ 11, 2016 7:39 pm

Αν a, b, c θετικοί πραγματικοί να αποδειχθεί ότι:

\dfrac{a}{b+c}+\dfrac{b}{c+a}+\dfrac{c}{a+b}+\dfrac{5a}{a+b}+\dfrac{5b}{b+c}+\dfrac{5c}{c+a}\geq 9


Houston, we have a problem!

Λέξεις Κλειδιά:
Άβαταρ μέλους
Ορέστης Λιγνός
Δημοσιεύσεις: 1835
Εγγραφή: Κυρ Μάιος 08, 2016 7:19 pm
Τοποθεσία: Χαλάνδρι Αττικής
Επικοινωνία:

Re: Ανισότητα!

#2

Μη αναγνωσμένη δημοσίευση από Ορέστης Λιγνός » Κυρ Δεκ 11, 2016 8:22 pm

Διονύσιος Αδαμόπουλος έγραψε:Αν a, b, c θετικοί πραγματικοί να αποδειχθεί ότι:

\dfrac{a}{b+c}+\dfrac{b}{c+a}+\dfrac{c}{a+b}+\dfrac{5a}{a+b}+\dfrac{5b}{b+c}+\dfrac{5c}{c+a}\geq 9


Διαγραφή λανθασμένης λύσης.
τελευταία επεξεργασία από Ορέστης Λιγνός σε Κυρ Δεκ 11, 2016 8:50 pm, έχει επεξεργασθεί 1 φορά συνολικά.


Κερδίζουμε ό,τι τολμούμε!
Άβαταρ μέλους
matha
Γενικός Συντονιστής
Δημοσιεύσεις: 6423
Εγγραφή: Παρ Μάιος 21, 2010 7:40 pm
Τοποθεσία: Θεσσαλονίκη

Re: Ανισότητα!

#3

Μη αναγνωσμένη δημοσίευση από matha » Κυρ Δεκ 11, 2016 8:34 pm

Ορέστης Λιγνός έγραψε: Αυτή γράφεται (a-b)(b-c)(a-c) \geq 0.

Έστω χωρίς βλάβη a \geq b \geq c.
Ορέστη, αυτό το επιχείρημα δεν είναι σωστό. Η αρχική ανισότητα είναι κυκλική, αλλά όχι συμμετρική. ως εκ τούτου δεν έχεις δικάιωμα να θεωρήσεις ότι δεν βλάπτεται η γενικότητα λέγοντας \displaystyle{a\geq b\geq c.}

Για να το πω και διαφορετικά, η ανισότητα
Ορέστης Λιγνός έγραψε: ...που ισοδυναμεί με a^2b+b^2c+c^2a \geq ab^2+bc^2+ca^2.
δεν ισχύει, όπως θα διαπιστώσεις αν θέσεις \displaystyle{a=1,b=2,c=3.}


Μάγκος Θάνος
Άβαταρ μέλους
matha
Γενικός Συντονιστής
Δημοσιεύσεις: 6423
Εγγραφή: Παρ Μάιος 21, 2010 7:40 pm
Τοποθεσία: Θεσσαλονίκη

Re: Ανισότητα!

#4

Μη αναγνωσμένη δημοσίευση από matha » Δευ Δεκ 12, 2016 7:36 am

Βάζω μια απόδειξη για την ανισότητα, από την οποία φαίνεται ότι το θέμα δεν απευθύνεται σε junior μαθητές.

Διώχνουμε τους παρονομαστές, οπότε έχουμε να αποδείξουμε ότι

\displaystyle{a^3+b^3+c^3+2(a^2b+b^2c+c^2a)\geq 3(ab^2+bc^2+ca^2).}

Λόγω κυκλικότητας μπορούμε να υποθέσουμε ότι \displaystyle{a=\min \{a,b,c\}}, οπότε υπάρχουν \displaystyle{x,y\geq 0,} ώστε \displaystyle{b=a+x,c=a+y.}

Με αντικατάσταση αυτών στην αποδεικτέα, αφού γίνουν οι πράξεις αναγόμαστε στην

\displaystyle{x^3+y^3+2x^2y-3xy^2+2a(x^2-xy+y^2)\geq 0.}

Επειδή φανερά ισχύει \displaystyle{2a(x^2-xy+y^2)\geq 0,} αρκεί να αποδείξουμε ότι \displaystyle{x^3+y^3+2x^2y-3xy^2\geq 0}.

Αν \displaystyle{y=0,} αυτή ισχύει. Διαφορετικά, διαιρώντας με \displaystyle{y^3} και θέτοντας \displaystyle{\frac{x}{y}=t\geq 0,} έχουμε να αποδείξουμε ότι

\displaystyle{t^3+1+2t^2\geq 3t}. Στο σημείο αυτό θα μπορούσαμε να συνεχίσουμε με λόγισμό (η συνάρτηση έχει θετική ελάχιστη τιμή στο διάστημα \displaystyle{[0,+\infty )}).

Μια στοιχειώδης απόδειξη είναι και η εξής:

\displaystyle{t^3+2t^2+1=t^3+t^2+t^2+\frac{1}{4}+\frac{1}{4}+\frac{1}{4}+\frac{1}{4}+\stackrel{AM-GM}{\geq }7\sqrt[7]{\frac{t^7}{4^4}}=\frac{7}{\sqrt[7]{4^4}}t}.

Πλέον αρκεί

\displaystyle{\frac{7}{\sqrt[7]{4^4}}>3}. Αυτό βλέπουμε ότι ισχύει, αφού οι γίνουν οι πράξεις ρουτίνας.


Μάγκος Θάνος
Άβαταρ μέλους
Demetres
Γενικός Συντονιστής
Δημοσιεύσεις: 8989
Εγγραφή: Δευ Ιαν 19, 2009 5:16 pm
Τοποθεσία: Λεμεσός/Πύλα
Επικοινωνία:

Re: Ανισότητα!

#5

Μη αναγνωσμένη δημοσίευση από Demetres » Δευ Δεκ 12, 2016 5:42 pm

matha έγραψε:Βάζω μια απόδειξη για την ανισότητα, από την οποία φαίνεται ότι το θέμα δεν απευθύνεται σε junior μαθητές.
Μεταφέρθηκε στους Seniors.


Άβαταρ μέλους
Διονύσιος Αδαμόπουλος
Δημοσιεύσεις: 807
Εγγραφή: Σάβ Μαρ 19, 2016 5:11 pm
Τοποθεσία: Πύργος Ηλείας

Re: Ανισότητα!

#6

Μη αναγνωσμένη δημοσίευση από Διονύσιος Αδαμόπουλος » Δευ Δεκ 12, 2016 7:58 pm

matha έγραψε:
Αρκεί να αποδείξουμε ότι \displaystyle{x^3+y^3+2x^2y-3xy^2\geq 0}
Μέχρι αυτό το σημείο συμφωνεί και η δικιά μου λύση. Από αυτό το σημείο και μετά:

Αν κάποιο από τα x, y είναι 0 ή x=y, τότε η ανισότητα ισχύει με ισότητα όταν x=y=0 (όταν δηλαδή οι a, b, c είναι ίσοι).

Θα χρησιμοποιηθεί παρακάτω η γνωστή ανισότητα: x^3+y^3>x^2y+xy^2 αν x, y διαφορετικοί θετικοί πραγματικοί.

Αν x>y>0, τότε έχουμε ότι:

x^3+y^3+2x^2y-3xy^2>x^3+y^3+2xy(x-y)-xy^2>x^3+y^3-xy^2>x^2y+xy^2-xy^2=x^2y>0

Αν y>x>0, τότε η ανισότητα μπορεί να γίνει (y-x)^3-(y-x)x^2+x^3>0 και θέτουμε y-x=k, όπου k>0.
Άρα έχουμε k^3+x^3-kx^2\geq kx^2+k^2x-kx^2=k^2x>0

Υ.Γ. Να προσθέσω πως κατασκεύασα αυτή την άσκηση προσπαθώντας να αποδείξω μια άλλη ανισότητα που σε κάποιο βήμα έφτανε στην:

a^3+b^3+c^3+2(a^2b+b^2c+c^2a) \geq 3(ab^2+bc^2+ca^2)

Είχα πραγματικά τρελαθεί :read: . Τόσο απλή (στην εμφάνιση) και να μην μπορεί να λυθεί με συνηθισμένες μεθόδους; Πιστεύοντας πως μάλλον χάνω κάτι και ότι υπάρχει πιο εύκολη λύση την έβαλα στους Juniors :roll: . Ευχαριστώ πάντως για την λύση!


Houston, we have a problem!
Απάντηση

Επιστροφή σε “Άλγεβρα - Προχωρημένο Επίπεδο (Seniors)”

Μέλη σε σύνδεση

Μέλη σε αυτήν τη Δ. Συζήτηση: Δεν υπάρχουν εγγεγραμμένα μέλη και 3 επισκέπτες